Tải bản đầy đủ (.pdf) (49 trang)

Định lý Roth, định lý Bertrand và một vài ứng dụng

Bạn đang xem bản rút gọn của tài liệu. Xem và tải ngay bản đầy đủ của tài liệu tại đây (353.11 KB, 49 trang )

Định lý Roth
Định lý Bertrand

Một vài ứng dụng

Vũ Thị Liễu
ĐH Thái Nguyên-ĐHKH
Ngày 16 tháng 04 năm 2015


Mục lục
1 Định đề Betrand
1.1 Số nguyên tố . . . . . . . . . . . . . . . . . . . .
1.2 Một vài cách biểu diễn số tự nhiên . . . . . . . .
1.3 Định đề Bertrand . . . . . . . . . . . . . . . . . .
2 Số Liouville và Định lý Roth
2.1 Số siêu việt Liouviile . . . . . . . . . . . . .
2.1.1 Tập đếm được, không đếm được . . .
2.1.2 Tập các số siêu việt . . . . . . . . . .
2.1.3 Xấp xỉ Diophante . . . . . . . . . . .
2.1.4 Số Liouville . . . . . . . . . . . . . .
2.2 Số siêu việt không là số Liouville . . . . . .
2.2.1 Tính siêu việt của số e . . . . . . . .
2.2.2 Tính siêu việt của số π . . . . . . . .
2.3 Giới thiệu Định lý Roth và vận dụng . . . .
2.3.1 Giới thiệu Định lý Roth . . . . . . .
2.3.2 Vận dụng Định lý Roth vào giải Toán
2.4 Một vài vận dụng vào giải Toán sơ cấp . . .

1


5
5
7
21

26
. . . 26
. . . 26
. . . 29
. . . 31
. . . 32
. . . 37
. . . 37
. . . 39
. . . 40
. . . 40
sơ cấp 42
. . . 43


Mở đầu
Cho đa thức f (x) = ad xd + ad−1 xd−1 + · · · + a1 x + a0 ∈ Z[x] với
a
ad > 0 và (ad , . . . , a1 , a0 ) = 1. Giả sử số hữu tỷ ∈ Q với b > 0.
b
Khi đó ta có biểu diễn dưới đây:
a
a
= ad
b

b

a
m
+ a0 = d , m ∈ Z.
b
b
a
m
a
= 0 hoặc f
= d với |m| 1.
Dễ dàng thấy ngay hoặc f
b
b
b
a
1
a
Như vậy, khi f
= 0 ta luôn có f
thỏa mãn cho
d
b
b
b
a
mọi số hữu tỷ với b > 0 và d = deg f (x). Một câu hỏi đầu tiên
b
có thể đặt ra: Liệu có thể thay thế số d bằng một số tự nhiên

a
a
dương s nào đó để với mỗi số hữu tỷ với b > 0 và f
= 0 ta
b
b
luôn có
a
1
f
?
b
bs
Giả sử α ∈ R \ Q và f (α) = 0. Khai triển Taylor của f (x) tại
x=α:
f

f (x) = f (α)+

d

+ ad−1

a
b

d−1

+ · · · + a1


f (α)
f (α)
f (d) (α)
(x−α)+
(x−α)2 +· · ·+
(x−α)d .
1!
2!
d!

Vì f (α) = 0 nên f

a
a
a
= b1 −α +b2 −α
b
b
b

2

+· · ·+bd

a
−α
b

d


f (i) (α)
với bi =
, i = 1, 2, . . . , d. Đặt r = max{|bi | | i = 1, . . . , d}.
i!
a
1
a
a
Khi
−α
1 ta luôn có d
f
rd − α
b
b
b
b
2


3

a
1
c(α)
có d <
− α . Bất đẳng thức này đã đưa
rd
b
b

đến một kết quả rất nổi tiếng của Liouville:
Giả sử α ∈ R là số vô tỷ đại số bậc d. Khi đó có hằng số c(α)
a
c(α)
a
để bất đẳng thức
− α ≥ d thỏa mãn cho mọi ∈ Q, b > 0.
b
b
b
Trong lý thuyết số, số Liouville là một số vô tỷ α sao cho
a
với mỗi số nguyên dương n, tồn tại số hữu tỷ với b > 1 sao
b
a
1
cho 0 < |α − | < n . Năm 1844, Liouville chứng minh rằng số
b
b
Liouville tồn tại và là số siêu việt. Kết quả này của Liouville là
xuất phát điểm cho định lý Roth hay định lý Thue-Siegel-Roth.
Định lý Roth phát biểu rằng với mỗi số đại số α ∈
/ Q và mỗi
số thực > 0, chỉ có hữu hạn cặp số nguyên (a, b) với b > 0
a
1
sao cho |α − | < 2+ . Định lý Roth cho thấy với mỗi số đại
b
b
số α cho trước, không thể có quá nhiều số hữu tỷ xấp xỉ đủ tốt

của α. Định lý Roth một kết quả cơ bản trong lý thuyết xấp xỉ
Diophante đối với các số đại số. Định lý Roth được cải tiến trong
suốt nửa thế kỷ, bắt đầu từ kết quả của Liouville năm 1844, của
Thue năm 1909, của Siegel năm 1921, của Dyson năm 1947 và của
Roth năm 1955. Với kết quả này Roth đã nhận được huy chương
Feilds.
Mục đích của luận văn là trình bày lại một số kết quả về số
Liouville, định lý Bertrand và định lý Roth.
Luận văn gồm 2 chương. Chương 1 dành để trình bày về Định đề
Bertrand. Chương 2 giới thiệu Định lý Roth và một số ứng dụng.
Chương thứ nhất gồm 3 mục. Mục 1.1 được dành để trình bày
về số nguyên tố. Mục 1.2 trình bày một vài cách biểu diễn số tự
nhiên. Mục 1.3 Chứng minh Định đề Bertrand. Chương thứ hai
gồm 4 mục. Mục 2.1 trình bày về số siêu việt Liouville. Mục 2.2
trình bày về số siêu việt không phải là số Liouville. Mục 2.3 nêu
lại nội dung Định lí Roth, trong đó gồm Bổ đề Siegel và Bổ đề tổ
hợp, song việc chứng minh Định lí Roth quá phức tạp nên chúng
tôi cũng chỉ trình bày lại kết quả mà không chứng minh. Mục 2.4
và với c(α) <


4

đã đưa ra áp dụng vào Toán sơ cấp. Luận văn được hoàn thành
dưới sự hướng dẫn của PGS.TS. Đàm Văn Nhỉ. Qua đây tôi xin
được bày tỏ lòng kính trọng và biết ơn sâu sắc đối với thầy hướng
dẫn, người đã tận tình chỉ bảo, quan tâm động viên và giúp đỡ tôi
hoàn thành bản luận văn này. Đồng thời tôi xin trân trọng cảm
ơn các thầy cô, các cán bộ khoa Toán Tin và các cán bộ quản lí
của trường Đại học Khoa học-Đại học Thái Nguyên đã dạy dỗ và

hết lòng giúp đỡ tôi trong suốt quá trình học tập tại trường.
Tôi xin gửi lời cảm ơn tới Sở Giáo dục- Đào tạo tỉnh Quảng
Ninh, Ban Giám hiệu trường THPT Vũ Văn Hiếu - Thành phố
Hạ Long, tỉnh Quảng Ninh đã tạo điều kiện cho tôi được hoàn
thành khóa học này.
Cuối cùng tôi xin gửi lời cảm ơn tới tập thể các bạn lớp Cao
học Toán K7Q trường ĐHKH- Đại học Thái Nguyên, đã nhiệt
tình giúp đỡ tôi trong quá trình học tập và quá trình hoàn thành
bản luận văn này.

Thái Nguyên, ngày 16 tháng 04 năm 2015.

Vũ Thị Liễu


Chương 1
Định đề Betrand
Trong chương này, chúng tôi giới thiệu một vài vấn đề nghiên cứu
của lý thuyết số. Bằng các kiến thức toán học sơ cấp, chúng ta
chỉ ra tập các số siêu việt không đếm được, cũng tìm được công
thức tính số siêu việt trong một vài trường hợp đặc biệt. Câu
chuyện về xây dựng số siêu việt hay một vài số đặc biệt khác còn
rất nhiều vấn đề thú vị cần khám phá.

1.1

Số nguyên tố

Định nghĩa 1.1.1. Số tự nhiên p > 1 không có một ước số dương
nào khác 1 và chính nó được gọi là số nguyên tố. Số tự nhiên q > 1

có ước số dương khác 1 và chính nó được gọi là hợp số. Nếu có
số tự nhiên d để n = d2 thì n được gọi là số chính phương.
Hiển nhiên ta có định lý sau đây:
Định lý 1.1.2. Cho số nguyên tố p và các số nguyên tuỳ ý m, a, b.
Khi đó
p nếu p | m
(i) (m, p) =
1 nếu p m.
(ii) Mọi số m > 1 đều có ước nguyên tố.
(iii) Nếu p | ab thì p | a hoặc p | b.
Ta thấy, trong đoạn [(n + 1)! + 2, (n + 1)! + n + 1] không có số
nguyên tố nào với mọi số nguyên dương n. Định lý sau đây chỉ ra
tập các số nguyên tố là một tập vô hạn.
5


6

Định lý 1.1.3. [Euclid] Tập hợp tất cả các số nguyên tố là một
tập vô hạn.
Ví dụ 1.1.4. Tồn tại nhiều vô hạn các số nguyên tố dạng 4n − 1
với n ∈ N.
Bài giải: Giả sử chỉ có một số hữu hạn các số nguyên tố p1 , . . . , ps
s

dạng 4n − 1. Đặt q = 4

pi − 1 > 1. Khi đó q là số lẻ. Nhận xét
i=1


(*) : Sử dụng quy nạp theo r ta dễ dàng chỉ ra tích

r

(4ni + 1)
i=1

các số nguyên dương dạng 4h + 1 cũng là một số nguyên dương
dạng 4m + 1. Nếu mọi ước nguyên tố của q đều có dạng 4k + 1
thì q phải có dạng 4m + 1. Vì q có dạng 4m − 1 nên q phải có
một ước nguyên tố p dạng 4k − 1. Từ điều giả sử ta suy ra p = pi
với i nào đó. Vậy p|(−1). Điều này không thể được. Như vậy có
nhiều vô hạn số nguyên tố dạng 4n − 1.
Định lý 1.1.5. Với mỗi số nguyên dương n đều tồn tại số nguyên
tố lớn hơn n.
Chứng minh: Xét số n! + 1. Khi chia số này cho các số nguyên
dương nhỏ hơn hoặc bằng n đều cho số dư là 1. Do vậy mọi ước
nguyên tố của n! + 1 đều không thuộc tập {1, 2, . . . , n} và như
thế nó phải lớn hơn n.
Định lý 1.1.6. [Định lý cơ bản của số học] Mọi số tự nhiên
lớn hơn 1 đều phân tích được thành một tích hữu hạn các thừa số
nguyên tố và sự phân tích này là duy nhất nếu không kể đến thứ
tự các thừa số.
Khi phân tích số tự nhiên q > 1 thành tích các thừa số nguyên
tố, có thể một số nguyên tố xuất hiện nhiều lần. Nếu các số
nguyên tố p1 , . . . , ps xuất hiện theo thứ tự α1 , . . . , αs lần, thì ta
viết
q = pα1 1 pα2 2 . . . pαs s
và ta gọi tích này là dạng phân tích chính tắc của q.



7

Cho hai số a, b có dạng phân tích chính tắc
a = pα1 1 pα2 2 . . . pαs s q1u1 . . . qrur , b = pβ1 1 pβ2 2 . . . pβs s tv11 . . . tvhh ,
trong đó các thừa số nguyên tố qi chỉ ở trong a, còn các thừa số
nguyên tố tj chỉ có trong b, khi đó ta có
min(α ,β )

2 2
ucln(a, b) = p1 min(α1 ,β1 ) p2
. . . ps min(αs ,βs ) ,
bcnn(a, b) = p1 max(α1 ,β1 ) p2 max(α2 ,β2 ) . . . ps max(αs ,βs ) q1 u1 . . . . . . th vh .

1.2

Một vài cách biểu diễn số tự nhiên

Ta đã biết cách biểu diễn một số tự nhiên theo cơ số 10. Định lý
sau cho cách biểu diễn một số nguyên dương theo cơ số k > 1 tuỳ
ý.
Định lý 1.2.1. Cho số nguyên dương k > 1. Mỗi số nguyên không
âm n đều có thể biểu diễn một cách duy nhất thành tổng
n = a0 k m + a1 k m−1 + · · · + am−1 k + am ,
trong đó a0 , . . . , am ∈ {0, 1, . . . , k − 1}.
Chứng minh: Sự tồn tại: Giả thiết n là số nguyên không âm.
Nếu 0
n
k − 1, ta đã có sự biểu diễn n = n. Nếu n
k

m
m+1
ta gọi m là số nguyên dương lớn nhất để k
n. Gọi
m
m
a0 ∈ {1, . . . , k − 1} để a0 k
n < (a0 + 1)k . Ta có 0 n1 = n −
m
m
a0 k < k . Tương tự trên, có ai , m − i để ai k m−i n1 < k m−i+1 .
Tương tự , sau một số hữu hạn bước ta có các ai ∈ {0, 1, . . . , k−1}
để n = a0 k m + a1 k m−1 + · · · + am−1 k + am .
Tính duy nhất: Giả sử ta có hai sự biểu diễn cho n theo k là
n = a0 k m + a1 k m−1 + · · · + am−1 k + am ,
n = b0 k s + b1 k s−1 + · · · + bs−1 k + bs .
Khi đó (a0 k m + · · · + am−1 k) − (b0 k s + · · · + bs−1 k) + (am − bs ) = 0.
Vì vế phải chia hết cho k nên am − bs chia hết cho k. Ta suy ra
am = bs . Lập luận tương tự, có m = s, ai = bi , ∀i, và sự biểu diễn
là duy nhất.


8

Với biểu diễn n = a0 k m + a1 k m−1 + · · · + am−1 k + am ta thường
viết
n = (a0 a1 · · · am−1 am )k .
Số k được gọi là cơ số, còn a0 , a1 , . . . , am được gọi là các chữ số
của biểu diễn n theo cơ số k và biểu diễn n = (a0 a1 · · · am−1 am )k

được gọi là biểu diễn k-phân.
Ngoài cách biểu diễn mỗi số nguyên theo một cơ số như trên
ta còn có nhiều cách biểu diễn khác nữa. Biểu diễn mỗi số nguyên
dương qua các tổ hợp được sử dụng trong đại số giao hoán.
Định lý 1.2.2. [Hilbert] Cho số nguyên dương d. Khi đó mỗi
số nguyên dương n đều có thể biểu diễn một cách duy nhất thành
tổng
hd
hd−1
h1
n=
+
+ ··· +
,
d
d−1
1
trong đó các số nguyên dương hi thoả mãn hd > hd−1 > · · · >
h1 0.
Chứng minh: Sự tồn tại: Giả thiết n là số nguyên dương. Ta
hd
chọn hd là số nguyên dương lớn nhất sao cho
n. Nếu
d
d
hd
hi
n=
thì n =
với hi = i − 1 cho i = 1, . . . , d − 1.

d
i
i=1
hd
Trong trường hợp n = n −
> 0, sử dụng giả thiết qui nạp
d
d−1 h
i
ta có thể coi n =
với hd−1 > hd−2 > · · · > h1 ≥ 0. Ta
i
i=1
hd + 1
còn phải chỉ ra hd > hd−1 . Thật vậy, từ
> n ta suy ra
d
hd
hd + 1
hd
hd−1
=

>n
. Vậy hd > hd−1 .
d
d
d−1
d−1
Tính duy nhất của biểu diễn được chứng minh bằng phương pháp

d
hi
qui nạp theo n. Chúng ta đã biết nếu n =
với hd >
i
i=1
hd
hd−1 > · · · > h1
0 thì hd là số lớn nhất thỏa mãn
n.
d


9

hd + 1
n.
d
d−1 h
hd + 1
hd
hd
hd−1 + 1
i
Khi đó

=
.
d−1
i

d
d
d−1
i=1
Điều này mâu thuẫn với giả thiết qui nạp và sự biểu diễn là duy
nhất.
Nếu n = 1 kết luận là hiển nhiên. Giả sử n > 1 và

Ví dụ 1.2.3. Với mỗi cặp số nguyên dương m, k, số m luôn có
sự biểu diễn duy nhất trong dạng
m=

hk
k

hk−1
hr
+ ··· +
,
k−1
r

+

ở đó các số nguyên không âm hi thỏa mãn hk > hk−1 > · · · >
hr r 1.
Ví dụ 1.2.4. Với mỗi cặp số nguyên dương m, k, ta luôn có
biểu diễn duy nhất k m =
n



k=1

ar,m Crk với các hệ số nguyên ar,m

r=1

m

km =

m

r=1

r+1
Cn+1
ar,m . Tìm ar,m .

m

m

Bài giải: Tồn tại biểu diễn duy nhất k =
n

lý 1.2.2. Ta có
k=1
r


Ta có ar,m =

km =

m

n

ar,m
r=1

Crk . Vậy

k=r

ar,m Crk theo Định

r=1
n

km =

k=1

m
r=1

Cr+1
n+1 ar,m .


(−1)r+i Cir im .

i=1

Định lý 1.2.5. [Cantor] Mỗi số nguyên dương a đều có thể biểu
diễn một cách duy nhất thành tổng
a = an n! + an−1 (n − 1)! + · · · + a2 2! + a1 1!
trong đó các số ai nguyên với 0

ai

i và an > 0.

Chứng minh: Ký hiệu pn = n!. Khi đó
1p1 + 2p2 + · · · + npn = pn+1 − 1.


10

Giả sử a = a1 p1 + a2 p2 + · · · + an pn với an = 0 và 0
1, . . . , n. Ta thấy ngay
b = a1 p1 +a2 p2 +· · ·+an−1 pn−1

ak

k, k =

1p1 +2p2 +· · ·+(n−1)pn−1 = pn −1.

Do đó b < pn . Mặt khác ta có pn

an p n
npn . Suy ra pn
an pn a < pn+1 . Vậy, cho mỗi số tự nhiên a có thể tìm thấy duy
nhất một số tự nhiên n để pn a < pn+1 . Từ bất đẳng thức này
suy ra tồn tại đúng một số an để an pn a < (an +1)pn . Từ đây ta
suy ra a = an pn +b với 0 b < pn . Lặp lại quá trình trên đối với b.
Từ đó suy ra sự biểu diễn a = an n!+an−1 (n−1)!+· · ·+a2 2!+a1 1!,
ở đó các ai nguyên với 0 ai i và an > 0. Tính duy nhất của
biểu diễn là hiển nhiên.
Định lý 1.2.6. Mỗi số hữu tỷ a ∈ (0; 1) đều có thể biểu diễn một
cách duy nhất thành tổng
a=

b1 b2
bn
+ + ··· +
,
2! 3!
(n + 1)!

ở đó các bi nguyên với 0

bi

i với mọi i và bn = 0.

Chứng minh: Bước 1: Chia đoạn [0; 1] ra làm hai đoạn bằng
1
nhau. Độ dài mỗi đoạn bằng . Bước 2: Chia mỗi đoạn nhỏ này
2!

1
ra ba phần bằng nhau. Độ dài mỗi đoạn bằng . Tiếp tục như
3!
vậy, bước thứ n : Chia mỗi đoạn có được từ bước thứ n − 1 ra
1
. Chú
làm n + 1 đoạn bằng nhau. Độ dài mỗi đoạn bằng
(n + 1)!
s
s.n!
ý rằng nếu a =
là phân số tối giản thì ta viết a =
.
n+1
(n + 1)!
m
Giả sử a =
với m không chia hết cho n + 1. Xét bước thứ
(n + 1)!
1
n để có đoạn với độ dài bằng
. Khi đó tồn tại số nguyên
(n + 1)!
an
m
an + 1
dương an để
<
. Gọi số nguyên dương

n!
(n + 1)!
n!
m
an
bn
bn thỏa mãn a =
=
+
. Lặp lại quá trình trên
(n + 1)!
n! (n + 1)!


11

an
an
b1 b2
bn−1
ta nhận được biểu diễn
= + +···+
, Tóm
n!
n!
2! 3!
n!
b1 b2
bn
lại a = + + · · · +

, ở đó các bi nguyên với 0 bi i
2! 3!
(n + 1)!
với mọi i và bn = 0.
cho số

Từ Định lý 1.2.5 và Định lý 1.2.6 ta suy ra ngay kết quả dưới
đây:
Ví dụ 1.2.7. Mỗi số hữu tỷ dương a đều có thể viết thành tổng
a = an n!+an−1 (n−1)!+· · ·+a2 2!+a1 1!+
ở đó các ai nguyên với 0
0 bi i.

ai

b1 b2
bm
+ +· · ·+
,
2! 3!
(m + 1)!

i, an > 0 và các bi nguyên với

Ví dụ 1.2.8. Xác định số tự nhiên dương a và b để (a3 +b)(b3 +a)
là lũy thừa của 3.
Bài giải: Hiển nhên mỗi số a3 + b và b3 + a đều phải là những
lũy thừa của 3. Biểu diễn a = 3k l, b = 3h m với l, m đều không
chia hết cho 3, còn k, h là những số nguyên không âm. Ta có thể
giả thiết k h. Ta có

a3 + b = 33k l3 + 3h m = 3h (33k−h l3 + m).
Để số a3 + b có thể bằng một lũy thừa của 3 cần 3k − h = 0. Từ
đó suy ra k = h = 0. Vậy a = l, b = m đều không chia hết cho 3.
Xét a ≡ 1(mod 3). Khi đó a3 ≡ 1(mod 3). Vậy b ≡ −1(mod 3).
Nếu a = 1 thì b = 3h − 1. Ta có a3 + b = 3h và b3 + a =
3h+1 [3h (3h−1 − 1) + 1]. Nếu h > 1 thì 3h (3h−1 − 1) + 1 không là lũy
thừa của 3. Do vậy h = 1 và a = 1.b = 2. Ta có (a3 +b)(b3 +a) = 33 .
Nếu a > 1 thì ta biểu diễn a = 3k l + 1, b = 3h m − 1 với các số
tự nhiên k, h, l, m và l, m đều không chia hết cho 3. Khi k
h
3
3k 3
2k+1 2
k+1
h
h
thì a + b = 3 l +
l +3
+ 3 m = 3 n với số tự nhiên
n > 1 và n không chia hết cho 3. Điều này không thể xảy ra. Khi
k < h thì b3 + a = 3k v với v > 1 và cũng không chia hét cho 3.
Điều này cũng không xảy ra. Lâp luận tương tự trên ta chỉ có
a = 2, b = 1.


12

Ví dụ 1.2.9. Chứng minh rằng, với bất kỳ số tự nhiên n > 20150
ta luôn luôn tìm được số tự nhiên m thỏa mãn


m = a2 + b2 , 0 < m − n < 3 4 n, với a, b ∈ N∗ .
Bài giải: Gọi x là số tự nhiên lớn nhất thỏa mãn x2
n <

2
2
(x + 1) . Vậy n − x
2x 2 n. Gọi y là số tự nhiên nhỏ nhất
2
2
thỏa mãn n − x < y . Vậy (y − 1)2 n − x2 < y 2 . Từ những việc
chọn này ta suy ra

y =y−1+1
n − x2 + 1
2 n + 1.
√ √
Do vậy y
2 4 n + 1. Ta chọn m = x2 + y 2 > n. Vậy m − n > 0
và m = x2 + y 2 với x, y ∈ N∗ . Ta
lại có m − n = y 2 − (n −


2
x√
)
y 2 − (y − 1)2 = 2y − 1
2 2 4 n + 1. Vì n > 20150 nên



2 2 4 n + 1 < 3 4 n. Như vậy, kết quả đã được chứng minh.
Ví dụ 1.2.10. Xác định số tự nhiên dương lớn nhất
b = a1 a2 . . . an−1 an
thỏa mãn 2a2 < a1 + a3 , 2a3 < a2 + a4 , . . . , 2an−1 < an−2 + an .
Bài giải: Đặt di = ai+1 − ai với i = 1, 2, . . . , n − 1. Hiển nhiên
ai − ai−1 < ai+1 − ai và suy ra di−1 di − 1. Giả sử am là chữ số
nhỏ nhất trong số các số a1 , . . . , an . Vì ai − ai−1 < ai+1 − ai nên ta
có thể coi am−1 > am , am am+1 . Do vậy dm−1 −1, dm 0. Khi
đó dm−2 dm−1 −1 −2, dm−3 −3, . . . ; còn dm+1 dm +1 1,
dm+2 2, dm+3 3, . . . .
Bây giờ ta chỉ ra: n m + 4. Thật vậy, nếu n > m + 4 thì ta có
am+5 = am+4 +dm+4 am+4 +4 am+3 +3+4 am+2 +2+3+4
am+1 + 10 10, mâu thuẫn vì am+5 9. Tương tự, ta chỉ ra m
4, vì trái lại sẽ có am−4 am−3 −am−4 am−3 +4 · · · am +10
(mâu thuẫn). Tóm lại n
8. Với n = 8 và khi đó m = 4 ta có
a1 , a8 9 và a2 = a1 + d1 9 − 3 = 6, a3 = a2 − 2 6 − 2 = 4,
a4 = a3 − 1 4 − 1 = 3; a7 = a8 − d7 9 − 3 = 6, a6 = a7 − 2
6 − 2 = 4, a5 = a6 − 1
4 − 1 = 3. Do vậy b
96433469 và
bln = 96433469.


13

Ví dụ 1.2.11. Với số tự nhiên a = an−1 an−2 . . . a1 a0 ta ký hiệu
số tự nhiên a∗ = a0 .10n−1 + a1 .10n−2 + · · · + an−2 .10 + an−1 . Với số
tự nhiên đương n hãy xác định số lớn nhất các số tự nhiên khác
nhau biểu diễn được thành dạng hiệu a − a∗ .

Bài giải: Ta thấy 1

an−1

9 và 0

a0 , a1 , . . . , an−2

9. Số

a−a∗ = d1 (10n−1 −1)+d2 (10n−2 −10)+· · ·+dk (10n−k −10k−1 ), (∗)
n
n−1
khi n chẵn, k =
khi n lẻ và ds = an−s − as−1 .
2
2
Như vậy, ta có hệ
ở đó k =

−8

d1

9, −9

ds

9, (∗∗)


với s = 2, 3, . . . , k. Vì hệ bất phương trình


−8 d1 9
−9 ds 9


d1 , ds ∈ Z, s = 2, 3, . . . , k
có nhiều nhất là 18.19k−1 nghiệm phân biệt nên số lớn nhất trong
các số tự nhiên khác nhau biểu diễn được thành dạng hiệu a − a∗
bằng
(i) 1 khi n = 1.
n
−1
(ii) 18.19 2 khi n chẵn.
n−3
(iii) 18.19 2 khi n lẻ.
Chú ý rằng, không khó khăn để chứng minh số biểu diễn dạng
(∗) là duy nhất thỏa mãn điều kiện (∗∗).
Ví dụ 1.2.12. [IMO 1978 Shortlisted Problems Fra 3]
Chứng minh rằng, với bất kỳ các số nguyên dương x, y, z thỏa
mãn xy − z 2 = 1 luôn có các số nguyên không âm a, b, c, d để
x = a2 + b2 , y = c2 + d2 và z = ac + bd.


14

Bài giải: Sử dụng kết quả về tính duy nhất của biểu diễn thành
tích các nhân tử bất khả quy trong Z[i] nên với bốn số m, n, p, q ∈
Z[i] thỏa mãn mn = pq thì có các số a, b, c, d ∈ Z[i] để m =

ab, n = cd, p = ac, q = bd. Vì vành Z[i] là vành nhân tử hóa nên
từ xy = z 2 + 1 = (z + i)(z − i) và từ đây có x = ab, y = cd, z + i =
ac, z − i = bd với a, b, c, d ∈ Z[i]. Biểu diễn a = a1 + ia2 , d =
d1 + id2 . Với b = a và c = d ta có x = a21 + a22 , y = d21 + d22 và
z = a1 d1 + a2 d2 .
Ví dụ 1.2.13. Chứng minh rằng nếu số nguyên dương m = a3 +
b3 + c3 với các số nguyên a, b, c thì m không thể biểu diễn dưới
dạng 9k + 4 hoặc 9k + 5.
Bài giải: Ta biết rằng, mỗi số tự nhiên n đều viết được thành
n = 3k + r với r = 0, 1, 2. Do vậy n3 = 9s hoặc 9s + 1 hoặc 9s + 8.
Ta có 1 = (9n4 )3 + (1 − 9n3 )3 + (3n − 9n4 )3 ; 2 = (1 + 6n3 )3 + (1 −
6n3 )3 +(−6n2 )3 ; 3 = 43 +43 +(−5)3 . Vậy m = a3 +b3 +c3 = 9t+u
với u ∈ {0, 1, 2, 3, 6, 7, 8}. Do vậy, m không thể biểu diễn thành
9k + 4 hoặc 9k + 5.
Ví dụ 1.2.14. Chứng minh rằng mỗi số nguyên dương m đều
biểu diễn được thành m = a3 + b3 + c3 + d3 + e3 với a, b, c, d, e ∈ Z.
Bài giải: Ta biết rằng, mỗi số tự nhiên n đều viết được thành
n = 6t + r với r = 0, 1, 2, 3, 4, 5. Từ r3 − r = (r − 1)r(r + 1) = 6k
suy ra 6t + r = (6t + r)3 + 6k. Do vậy n = 6t + r = (6t + r)3 + 6k =
(6t + r)3 + (k + 1)3 + (k − 1)3 + (−k)3 + (−k)3 .
Ví dụ 1.2.15. Chứng minh rằng, mỗi số nguyên dương m đều
biểu diễn được thành tổng các số Fn , trong đó (Fn ) là dãy Fibonacci cho bởi F1 = F2 = 1, Fn+2 = Fn+1 + Fn với n 1.
Bài giải: Bằng quy nạp theo m ta có ngay kết quả.
Ví dụ 1.2.16. [IMO 1998] Giả sử hàm số học f xác định trên
tập N thỏa mãn f (1) = 1, f (3) = 3 và với mỗi số nguyên dương n
ta đều có f (2n) = f (n), f (4n+1) = 2f (2n+1)−f (n)f (4n+3) =
3f (2n + 1) − 2f (n). Hãy xác định số tất cả các số nguyên dương
n thỏa mãn n < 2015 và f (n) = n.



15

Bài giải: Đầu tiên ta chỉ ra: Nếu biểu diễn theo cơ số 2 của số n
là n = b1 b2 . . . bk(2) với b1 = 1, thì f (n) = bk bk−1 . . . b1 (2) , trong đó
xem 0bk−1 . . . b1 = bk−1 . . . b1 khi bk = 0. Thật vậy, với n = 1 = 1
có f (1) = 1; với n = 2 = 10 có f (2) = f (1) = 1 = 1 = 01; với
n = 3 = 11 có f (3) = 3 = 11. Quy nạp theo n. Giả sử kết luận
đúng với tất cả các số tự nhiên nhỏ hơn k + 1.
Ví dụ 1.2.17. [Lucas] Cho hai số nguyên không âm m, n và số
nguyên tố p. Biểu diễn
m = mr pr + · · · + m1 p + m0
n = nr pr + · · · + n1 p + n0
với 0

mi , ni < p, i = 0, 1, . . . , r. Chứng minh hệ thức sau đây
m
n



mr
m1
···
nr
n1

m0
(mod p).
n0
r


Bài giải: Bởi vì (1 + x)m = (1 + x)mr p
nhận được đồng nhất thức

+···+m1 p+m0

nên chúng ta

r

(1 + x)m = (1 + x)mr p . . . (1 + x)m1 p (1 + x)m0 .
Từ đó suy ra
r

(1 + x)m ≡ (1 + xp )mr . . . (1 + xp )m1 (1 + x)m0 (mod p).
Tính hệ số của xn ở hai vế của đồng nhất thức này ta nhận được
hệ thức
m
mr
m1
m0

···
(mod p).
n
nr
n1
n0
Ví dụ 1.2.18. Cho số nguyên dương n. Xác định ước số chung
lớn nhất d của các số

2n
2n
2n
,
,...,
.
1
3
2n − 1


16
n

Bài giải: Bởi vì 0 = (1 − 1)

2n
2k − 1

=

2c+1
2k − 1

=

=

2n


2k

n

2n
nên
2k − 1
k=0
k=1
n
2n
2n−1
chúng ta nhận được đồng nhất thức 2
=
. Do
k=1 2k − 1
2n
2n
2n
vậy, ước chung lớn nhất của các số
,
,...,
1
3
2n − 1
2n−1
a
phải là một ước của 2
và suy ra d = 2 với một số nguyên
không âm a nào đó. Vì d là một ước của 2n

nên d|2n. Giả sử
1
c
n = 2 , ở đó là số tự nhiên lẻ. Ta sẽ chỉ ra d = 2c+1 . Thật vậy,
ta có
2n

2c+1 − 1
2c+1
.
2k − 1 2k − 2

2c+1 − 1
Vì 2k − 1 và 2
là nguyên tố cùng nhau nên
2k − 1 2k − 2
2n
là số nguyên. Do vậy 2c+1 là một ước của
. Từ đó suy
2k − 1
ra d = 2c+1 .
c+1

Ví dụ 1.2.19. [Spania 1977] Cho các số nguyên a1 , a2 , . . . , an
n

ai = 1. Chứng minh rằng các số bi = ai + 2ai+1 3ai+2 +

với tổng
i=1


· · · + (n − i + 1)an + (n − i + 2)a1 + (n − i + 3)a2 + · · · + nai−1 với
i = 1, 2, . . . , n không có hai số nào trùng nhau.
Bài giải: Biến đổi bi − bi+1 = (1 − n)ai + ai+1 + ai+2 + · · · + an +
a1 + a2 + · · · + ai−1 = 1 − nai ≡ 1(mod n) với i = 1, 2, . . . , n − 1.
Như vậy
b1 ≡ b2 + 1(mod n)
b2 ≡ b3 + 1(mod n)
· · · ≡ · · · (mod n)
bn−1 ≡ bn + 1(mod n)
và nhận được bn−i ≡ bn + i(mod n) với i = 0, 1, 2, . . . , n − 1. Từ


17

đây suy ra rằng, các số bi chia cho n có các phần dư khác nhau.
Do đó, các số bi là phân biệt đôi một.
Ví dụ 1.2.20. Với mỗi số nguyên dương n, ký hiệu δ(n) là tổng
các bình phương nghịch đảo của các ước dương của n. Chứng
minh rằng, δ là hàm nhân và nếu n với phân tích tiêu chuẩn
n = pα1 1 pα2 2 . . . pαs s thì
δ(n) =
d|n

1
1
=
d2
n2


Bài giải: Hàm δ : N∗ → Q, n →

s

2(αi +1)

pi

i=1

−1
.
p2i − 1

1
, là hàm nhân. Áp dụng công
n2

thức tổng trải có
δ(n) =
d|n

1
=
d2

αi

s


p−2j
i )

(1 +
i=1

j=1

1
= 2
n

s

2(αi +1)

pi

i=1

p2i

−1
.
−1

Ví dụ 1.2.21. Cho a, b ∈ N∗ . Ký hiệu
L(a, b) = {xa + yb|x, y ∈ N}.
Với số nguyên dương a1 , a2 , a3 đôi một nguyên tố cùng nhau, đặt
l1 = min{l ∈ N∗ |la1 ∈ L(a2 , a3 )}

l2 = min{l ∈ N∗ |la2 ∈ L(a3 , a1 )}
l3 = min{l ∈ N∗ |la3 ∈ L(a1 , a2 )}.
Giả sử l1 , l2 , l3

2. Khi đó tồn tại biểu diễn duy nhất
l1 a1 = n12 a2 + n13 a3 , n12 , n13 ∈ N
l2 a2 = n21 a1 + n23 a3 , n21 , n23 ∈ N
l3 a3 = n31 a1 + n32 a2 , n31 , n32 ∈ N,

trong đó các hệ số nij
1 với mọi i = j và l1 = n21 + n31 , l2 =
n12 + n32 , l3 = n13 + n23 .


18

Bài giải: Ta có biểu diễn l1 a1 = n12 a2 + n13 a3 , n12 , n13 ∈ N;
l2 a2 = n21 a1 +n23 a3 , n21 , n23 ∈ N và l3 a3 = n31 a1 +n32 a2 , n31 , n32 ∈
N, theo định nghĩa. Bây giờ ta chỉ ra nij 1 khi i = j.
Thật vậy, giả sử có cặp i, j để nij = 0. Không làm mất tính chất
tổng quát, ta xét chẳng hạn n13 = 0. Khi đó l1 a1 = n12 a2 . Vì
(a1 , a2 ) = 1, l1 2, nên a2 chia hết l1 . Vậy l1 a2 . Vì l3 2 nên
a3 < l3 a3 = n31 a1 + n32 a2 , n31 , n32 ∈ N, và suy ra a3 ∈ M (a1 , a2 ).
Ta có a3 G(a1 , a2 )−1 = a1 a2 −a1 −a2 . Vì (a1 , a2 ) = 1 nên có hai
số nguyên p, q thỏa mãn pa1 +qa2 = 1. Từ quan hệ a1 a2 −a1 −a2
a3 = a1 a3 p + a2 a3 q và bằng a1 a2 + (a3 p − a2 )a1 + a2 a3 q suy ra
(a3 p − a2 + 1)a1 + a2 (a3 q + 1)
0. Dễ dàng thấy a3 p − a2 < 0
hoặc a3 q < 0. Vì thế, bằng cách thêm bớt a1 a2 , nếu cần thiết, để
có biểu diễn a3 = a1 a2 − k1 a1 − k2 a2 với các số nguyên k1 , k2 1.

Ta có ngay (a2 − k1 )a1 = k2 a2 + a3 . Vì k2
1 nên a2 − k1 > 0.
Vậy a2 − k1
l1 theo định nghĩa của l1 và nhận được l1 < a2 :
mâu thuẫn với điều kiện l1 a2 ở trên. Vậy nij 1 khi i = j.
Đặt γ1 = l1 − n21 − n31 , γ2 = l2 − n12 − n32 , γ3 = l3 − n13 − n23 .
Xét tổng T = γ1 a1 + γ2 a2 + γ3 a3 . Ta có
T =
+
=
+

(l1 − n21 − n31 )a1 + (l2 − n12 − n32 )a2
(l3 − n13 − n23 )a3
(l1 a1 − n12 a2 − n13 a3 ) + (l2 a2 − n21 a1 − n23 a3 )
(l3 a3 − n31 a1 − n32 a2 ) = 0.

Ta nhận được tổng γ1 a1 + γ2 a2 + γ3 a3 = 0. Bây giờ ta chứng minh
γ1 = γ2 = γ3 = 0 bằng phương pháp phản chứng. Giả sử γ1 , γ2 , γ3
không đồng thời bằng 0. Vì a1 , a2 , a3 > 0 nên không thể xảy ra
khả năng ba số γ1 , γ2 , γ3 cùng dấu. Vậy, ta chỉ cần xét hai trường
hợp sau:
(i) γ1 > 0, γ2 < 0, γ3

0.

(ii) γ1 > 0, γ2 > 0, γ3 < 0.
Xét trường hợp (i): Từ quan hệ γ1 a1 + γ2 a2 + γ3 a3 = 0 ta có biến
đổi
(l1 −n21 −n31 )a1 = γ1 a1 = −γ2 a2 −γ3 a3 = (−γ2 )a2 +(−γ3 )a3 > 0.



19

Từ đây suy ra (l1 − n21 − n31 )a1 ∈ L(a2 , a3 ). Theo định nghĩa của
l1 có l1 − n21 − n31
l1 với n21 , n31
1, nhưng điều này không
xảy ra.
Xét trường hợp (ii): Từ quan hệ γ1 a1 + γ2 a2 + γ3 a3 = 0 có thể
viết
(n13 + n23 − l3 )a3 = (−γ3 )a3 = γ1 a1 + γ2 a2
Từ đây suy ra n13 + n23 − l3 > 0 và (n13 + n23 − l3 )a3 ∈ L(a1 , a2 ).
Theo định nghĩa của l3 ta có n13 + n23 − l3 l3 hay n13 + n23
2l3 . Vậy, hoặc n13
l3 hoặc n23
l3 ; chẳng hạn: n13
l3 . Vì
l1 a1 = n12 a2 + n13 a3 , l3 a3 = n31 a1 + n32 a2 nên
(l1 − n31 )a1 = (n12 + n32 )a2 + (n13 − l3 )a3 ∈ L(a2 , a3 ).
Vậy l1 −n31 > 0 và l1 −n31 l1 với n31 1. Từ điều vô lý này suy
ra γ1 = γ2 = γ3 = 0 hay l1 = n21 +n31 , l2 = n12 +n32 , l3 = n13 +n23 .
Tính duy nhất của biểu diễn li ai là hiển nhiên và ta có (i).
Ví dụ 1.2.22. [Fibonacci] Xét dãy a0 = a1 = 1 và an+1 =
an + an−1 với n 1. Khi đó
(1) an+1 an−1 − a2n = (−1)n+1 .
(2) a4n = an+1 an−1 an+2 an−2 + 1.
(3) an + 1 là hợp số khi n
Bài giải: (1) Biểu diễn


3.
an+1
an

=

1 1
1 0
n
1 1
1 0

an+1
1
=
an
1
n−1
1 1
a b
Đặt
=
.
1 0
c d
a+b
an+1
an
Ta được
=


=
c+d
an
an−1
2a + b a + b
a
an+1 an
=
=
Vậy
2c + d c + d
c
an an−1

Như vậy có biểu diễn

an
an−1
,n

.
1.

2a + b
.
2c + d
b
1 1
d

1 0

2

.


20
n+1

an+1 an
1 1
hay
=
.
an an−1
1 0
Lấy định thức ở hai vế để có đồng nhất thức
an+1 an−1 − a2n = (−1)n+1 .
(2) Hiển nhiên có an+2 = 3an−1 + 2an−2 và an+1 = 2an−1 + an−2 .
Vậy
an+2 an−2 − an+1 an−1 =
=
=
=
=

(3an−1 + 2an−2 )an−2 − an+1 an−1
2a2n−2 + an−1 [3an−2 − an+1 ]
2a2n−2 + an−1 [2an−2 − 2an−1 ]

−2a2n−1 + an−2 [2an−2 + 2an−1 ]
2an−2 an − 2a2n−1 = 2(−1)n . Do đó

a4n = [an+1 an−1 +(−1)n ][an+2 an−2 −(−1)n ] = an+1 an−1 an+2 an−2 +1.
(3) Nếu an + 1 = p là số nguyên tố, n 3, thì từ hệ thức
a4n = an+1 an−1 an+2 an−2 + 1
suy ra an+1 an−1 an+2 an−2 ˙: p. Vậy an+1 an+2 ˙: p. Nếu an+1 ˙: p thì
an−1 ˙: p : mâu thuẫn. Nếu an+2 ˙: p thì an+1 − 1 + an + 1 ˙: an + 1
hay an+1 − 1 ˙: an + 1. Vậy an−1 − 2 ˙: an + 1. mâu thuẫn vì n 3.
Do đó an + 1 là hợp số.
Ví dụ 1.2.23. [Dãy Lucas] Xét dãy số Lucas (Ln ) xác định bởi
L0 = 2, L1 = 1 và Ln+1 = Ln + Ln−1 với mọi n 1. Chứng minh
(1) L2n = L2n − 2(−1)n và L2n−1 = Ln Ln−1 − (−1)n .
(2) a2n = an Ln , ở đó (an ) là dãy số Fibonacci.
(3) Ln+1 Ln−1 = L2n + 5(−1)n−1 .


1− 5
1+ 5
Bài giải: Với t1 =
và t2 =
, bằng quy nạp theo n
2
2
ta nhận được biểu diễn Ln = tn2 + tn1 . Các kết quả (1), (2) và (3)
được suy ra từ công thức tính Ln .


21


1.3

Định đề Bertrand

Chúng ta biết rằng dãy các số nguyên tố 2, 3, 5, 7, ... là vô hạn.
Người ta muốn kiểm tra khoảng cách giữa hai số nguyên tố liên
tiếp là không bị chặn: Với một số nguyên dương k tùy ý, luôn tồn
tại hai số nguyên tố kề nhau có hiệu vượt quá k. Để chứng minh
điều này, ta chỉ cần chỉ ra tồn tại k số tự nhiên liên tiếp không
là số nguyên tố. Thật vậy, nếu ta đặt N = 2.3.5....p là tích tất
cả các số nguyên tố không vượt quá k + 2 thì không số nào trong
dãy gồm k số nguyên liên tiếp
N + 2, N + 3, . . . , N + (k + 1)
là số nguyên tố. Nhưng vẫn có chặn trên cho khoảng cách trong
dãy các số nguyên tố. Năm 1845, Bertrand đưa ra giả thuyết,
thường gọi là Định đề Bertrand (Bertrand’s postulate), phát
biểu như sau:
Cho số nguyên dương n
p 2n.

1, luôn có số nguyên tố p với n <

Bertrand đã chứng minh giả thuyết trên cho n < 3000000. Năm
1850, Tchebychev là người đầu tiên đưa ra một chứng minh bằng
giải tích cho định đề trên. Năm 1932, Paul Erd˝os đã đưa ra một
chứng minh đẹp cho Định đề Bertrand chỉ dùng các kiến thức
Toán sơ cấp, khi ông mới 19 tuổi. Tư tưởng chính của Erd˝os
trong việc chứng minh Định đề Bertrand chính là việc ước lượng
2n
giá trị

. Trong mục này, chúng tôi sẽ giới thiệu đến các bạn
n
đọc chứng minh của Erd˝os cùng một vài số kết quả gần đây về
vấn đề này.
Ta thấy rằng, với n 2 ta có
2n
n
Thật vậy,

2n
n

4n
.
2n

là số hạng lớn nhất trong 2n số hạng

2n
+
0


22

2n
2n
2n
,
,...,

2n
1
2n − 1
2n

i=0

2n
i

của tổng
= (1 + 1)2n = 4n .

Erd˝os đã chứng minh nếu không có số nguyên tố p sao cho n <
2n
4n
trừ một số giá trị n
p
2n thì chúng ta sẽ suy ra
<
2n
n
nhỏ. Điều này chứng minh được Định đề Bertrand cho số n đủ
lớn. Đối với giá trị n nhỏ, Erd˝os kiểm tra trực tiếp. Để chứng
minh các ý tưởng trên, chúng ta cần một số bổ đề sau:
Bổ đề 1.3.1. Định đề Bertrand đúng với n < 4000.
Chứng minh: Để chứng minh điều này, ta không cần kiểm tra
tất cả 4000 trường hợp mà chỉ cần kiểm tra,
2, 3, 5, 7, 13, 23, 43, 83, 163, 317, 631, 1259, 2503, 4001,
là dãy các số nguyên tố mà mỗi số đứng sau không vượt quá 2

lần số đứng ngay đằng trước nó. Do đó, mọi khoảng {m : n <
m 2n} với n < 4000 đều chứa đựng một trong 14 số nguyên tố
của dãy trên.
Bổ đề 1.3.2. [Định lý Lagrange] Cho số nguyên tố p và số tự
nhiên n. Ký hiệu vp (n) là số mũ lớn nhất của p sao cho pvp (n) | n.
Khi đó
n
vp (n!) =
[ i ].
p
i 1
Bổ đề 1.3.3. Nếu p |

2n
n

2n

thì pvp (( n ))

2n.

Chứng minh: Đặt r(p) là số tự nhiên thỏa mãn pr(p)
2n
r(p)
r(p)
2n
n
=
v

((2n)!)

2v
(n!)
=
[
]

2
[
].
pr(p)+1 . Ta có vp 2n
p
p
n
i
i
i=1 p
i=1 p
r(p)
2n
2n
n
2n
Do vậy vp n =
([ i ] − 2[ i ])
r(p). Như vậy pvp (( n ))
p
i=1 p
r(p)

p
2n.


23


Từ bổ đề trên ta thấy số nguyên tố p > 2n xuất hiện trong sự
2n
phân tích của
nhiều nhất một lần. Hơn nữa, số nguyên tố
n
2n
2n
< p ≤ n sẽ không là ước của
p thỏa mãn
với n 3.
3
n
Bổ đề 1.3.4. Với mọi số thực x

2, ta có

p

4x−1 (∗), trong

p x

đó tích chạy qua tất cả các số nguyên tố p.

Chứng minh: Nếu q là số nguyên tố lớn nhất thỏa mãn q
thì
p=
p và 4q−1 4x−1 .
p x

x

p q

Do đó ta chỉ cần chứng minh cho trường hợp x = q là số nguyên
tố bằng quy nạp. Với q = 2, (*) hiển nhiên đúng. Bây giờ ta xem
xét (*) cho trường hợp số nguyên tố lẻ q = 2m + 1. Giả sử (*)
đúng cho tất cả các số tự nhiên x 2m. Cho q = 2m + 1, ta có
p=
p 2m+1

p
p m+1

p.
m+1


(2m + 1)!
2m + 1

là số tự
m!(m + 1)!
m
m+1


nhiên và tất cả các số nguyên tố ở vế trái là ước của tử số (2m+1)!
nhưng không phải là ước của mẫu số m!(m + 1)!. Bởi vì
p

Hiển nhiên

2m + 1
m
nên

2m + 1
m

=

2m + 1
m+1

2m+1


k=0

22m . Do đó

p
p 2m+1

2m + 1
k

4m

= 22m+1

2m + 1
m

4m 22m =

2m

4 .
Định lý 1.3.5. [Bertrand’s postulate] Cho số nguyên dương
n 1, luôn có số nguyên tố p với n < p 2n.


24

2n
. Từ (*) và Bổ đề
n

Chứng minh: Trước hết ta đánh giá
4n
3, ta có
2n

1.3.3, với n
hay


4

n

2n
n

2n
p


1+ 2n

(2n)

2n
Kết hợp với Bổ đề 1.3.2, ta được 4n


n

(2n)1+

2n

p


43




2n
3

p


2n
3

2n
p,
n


p.
n


1+ 2n

(2n)

2n

p hay

43


n

2n

p(∗∗). Giả sử rằng tồn tại số nguyên dương
n


n sao cho không có số nguyên tố p thỏa mãn n < p
Bổ đề 1.3.1, ta suy ra n
4000. Từ (**), ta có

1+ 2n

n
3


3(1+ 2n)

2n

2n. Theo
p = 1

n


và 4
(2n)
hay 2
(2n)
. Dùng bất đẳng thức


a + 1 < 2a , a 2, ta có




6
6
6
6
6[ 6 2n]
6 6 2n
2
.
2n = ( 2n) < ([ 2n] + 1) < 2
2n


3(1+ 2n)


6


2n(18+18 2n)

√ √
20 6 2n 2n

2/3


<2
<2
= 220(2n)
Do đó 2
(2n)
với n
4000. Từ đó suy ra (2n)1/3 < 20 hay n < 4000 (mâu
thuẫn). Vậy ta chứng minh xong Định đề Bertrand.
Từ Định đề Bertrand, Greenfield và Greenfield đã chỉ ra một hệ
quả khá đẹp như sau:
Hệ quả 1.3.6. Với số nguyên dương n, tập {1, 2, . . . , 2n} được
chia thành các cặp {a1 , b1 }, . . . , {an , bn } sao cho với mỗi 1 i
n, ai + bi là số nguyên tố.
Chứng minh: Ta chứng minh quy nạp theo n. Với n = 1, kết
quả là tầm thường. Với n > 1, đặt p là số nguyên tố thỏa mãn
2n < p 4n. Từ 4n không là số nguyên tố nên p = 2n + m với
1
m < 2n và m là số lẻ. Rõ ràng tập {m, . . . , 2n} được chia
2n − m + 1
thành
cặp
2
m−1
m+1
(m, 2n), (m + 1, 2n − 1), ..., n +
,n +
2
2



×